Homotopy of continuous map from a space with finite fundamental group












1












$begingroup$


Problem source: 2c on the UMD January, 2018 topology qualifying exam, seen here https://www-math.umd.edu/images/pdfs/quals/Topology/Topology-January-2018.pdf I have an argument for it, but I am not at all sure that it is correct.
In particular, I would like to know 1) if it is correct and 2) if there is a better way of doing it (especially if my way is incorrect).



Let X be a connected CW complex with $pi_1(X)$ finite. Let Y be a CW complex. Let $f:Xto Ytimes S^1$ be a continuous map. Show there exists a $g:Xto Y$ such that $f$ is homotopic to the map $(g,1):Xto Ytimes S^1$.



I am fairly confident in the first part of the proof, ie that $|f_*pi_1(X)|<infty$ (although my group theory is rusty). Since $f$ is continuous, it induces a group homomorphism on $pi_1(X)$, so 1) $f_*pi_1(X)simeqpi_1(X)/Ker(f_*)$ and 2) $f_*pi_1(X)leqpi_1(Ytimes S^1)simeqpi_1(Y)times mathbb{Z}$. In particular, (1) gives that $f_*pi_1(X)$ is finite, hence $simeq Htimes {0}$, where $H$ is some subgroup of $pi_1(Y)$, by (2).



The part I am much less sure of (that it is either correct or necessary) is the remainder. Take $p:Ytimesmathbb{R}to Ytimes S^1$ by $p(y,x)=(y,e^{2pi ix})$. This is a covering map, and $p_*pi_1(Ytimesmathbb{R})=pi_1(Y)times{0}$, so $f$ lifts to some $tilde{f}$. Since the image of any closed loop in $X$ is taken to a closed loop that does not wind around $S^1$ (else, we would get a generator for an infinite fundamental group), $tilde{f}$ is homotopic to some $(g,0)$, which maps to $(g,1)$ under $p$.



I have not used the fact that X and Y are CW complexes anywhere, but the only theorem that I know offhand in the context of CW complexes, subsets, and homotopy of functions would be the Homotopy Extension Theorem, which wouldn't make sense here (since $f(X)$ is not necessarily a CW complex).










share|cite|improve this question











$endgroup$












  • $begingroup$
    The map to $Y$ is a red herring, since maps into a product are determined by the maps into the factors. For a fancy proof using cohomology/homology see: math.stackexchange.com/questions/380383/… .
    $endgroup$
    – Justin Young
    Dec 30 '18 at 13:30










  • $begingroup$
    That's a very nice proof, although I'll need to make it to homology in my notes before I can properly parse what's going on.
    $endgroup$
    – Pepper
    Dec 31 '18 at 14:10


















1












$begingroup$


Problem source: 2c on the UMD January, 2018 topology qualifying exam, seen here https://www-math.umd.edu/images/pdfs/quals/Topology/Topology-January-2018.pdf I have an argument for it, but I am not at all sure that it is correct.
In particular, I would like to know 1) if it is correct and 2) if there is a better way of doing it (especially if my way is incorrect).



Let X be a connected CW complex with $pi_1(X)$ finite. Let Y be a CW complex. Let $f:Xto Ytimes S^1$ be a continuous map. Show there exists a $g:Xto Y$ such that $f$ is homotopic to the map $(g,1):Xto Ytimes S^1$.



I am fairly confident in the first part of the proof, ie that $|f_*pi_1(X)|<infty$ (although my group theory is rusty). Since $f$ is continuous, it induces a group homomorphism on $pi_1(X)$, so 1) $f_*pi_1(X)simeqpi_1(X)/Ker(f_*)$ and 2) $f_*pi_1(X)leqpi_1(Ytimes S^1)simeqpi_1(Y)times mathbb{Z}$. In particular, (1) gives that $f_*pi_1(X)$ is finite, hence $simeq Htimes {0}$, where $H$ is some subgroup of $pi_1(Y)$, by (2).



The part I am much less sure of (that it is either correct or necessary) is the remainder. Take $p:Ytimesmathbb{R}to Ytimes S^1$ by $p(y,x)=(y,e^{2pi ix})$. This is a covering map, and $p_*pi_1(Ytimesmathbb{R})=pi_1(Y)times{0}$, so $f$ lifts to some $tilde{f}$. Since the image of any closed loop in $X$ is taken to a closed loop that does not wind around $S^1$ (else, we would get a generator for an infinite fundamental group), $tilde{f}$ is homotopic to some $(g,0)$, which maps to $(g,1)$ under $p$.



I have not used the fact that X and Y are CW complexes anywhere, but the only theorem that I know offhand in the context of CW complexes, subsets, and homotopy of functions would be the Homotopy Extension Theorem, which wouldn't make sense here (since $f(X)$ is not necessarily a CW complex).










share|cite|improve this question











$endgroup$












  • $begingroup$
    The map to $Y$ is a red herring, since maps into a product are determined by the maps into the factors. For a fancy proof using cohomology/homology see: math.stackexchange.com/questions/380383/… .
    $endgroup$
    – Justin Young
    Dec 30 '18 at 13:30










  • $begingroup$
    That's a very nice proof, although I'll need to make it to homology in my notes before I can properly parse what's going on.
    $endgroup$
    – Pepper
    Dec 31 '18 at 14:10
















1












1








1





$begingroup$


Problem source: 2c on the UMD January, 2018 topology qualifying exam, seen here https://www-math.umd.edu/images/pdfs/quals/Topology/Topology-January-2018.pdf I have an argument for it, but I am not at all sure that it is correct.
In particular, I would like to know 1) if it is correct and 2) if there is a better way of doing it (especially if my way is incorrect).



Let X be a connected CW complex with $pi_1(X)$ finite. Let Y be a CW complex. Let $f:Xto Ytimes S^1$ be a continuous map. Show there exists a $g:Xto Y$ such that $f$ is homotopic to the map $(g,1):Xto Ytimes S^1$.



I am fairly confident in the first part of the proof, ie that $|f_*pi_1(X)|<infty$ (although my group theory is rusty). Since $f$ is continuous, it induces a group homomorphism on $pi_1(X)$, so 1) $f_*pi_1(X)simeqpi_1(X)/Ker(f_*)$ and 2) $f_*pi_1(X)leqpi_1(Ytimes S^1)simeqpi_1(Y)times mathbb{Z}$. In particular, (1) gives that $f_*pi_1(X)$ is finite, hence $simeq Htimes {0}$, where $H$ is some subgroup of $pi_1(Y)$, by (2).



The part I am much less sure of (that it is either correct or necessary) is the remainder. Take $p:Ytimesmathbb{R}to Ytimes S^1$ by $p(y,x)=(y,e^{2pi ix})$. This is a covering map, and $p_*pi_1(Ytimesmathbb{R})=pi_1(Y)times{0}$, so $f$ lifts to some $tilde{f}$. Since the image of any closed loop in $X$ is taken to a closed loop that does not wind around $S^1$ (else, we would get a generator for an infinite fundamental group), $tilde{f}$ is homotopic to some $(g,0)$, which maps to $(g,1)$ under $p$.



I have not used the fact that X and Y are CW complexes anywhere, but the only theorem that I know offhand in the context of CW complexes, subsets, and homotopy of functions would be the Homotopy Extension Theorem, which wouldn't make sense here (since $f(X)$ is not necessarily a CW complex).










share|cite|improve this question











$endgroup$




Problem source: 2c on the UMD January, 2018 topology qualifying exam, seen here https://www-math.umd.edu/images/pdfs/quals/Topology/Topology-January-2018.pdf I have an argument for it, but I am not at all sure that it is correct.
In particular, I would like to know 1) if it is correct and 2) if there is a better way of doing it (especially if my way is incorrect).



Let X be a connected CW complex with $pi_1(X)$ finite. Let Y be a CW complex. Let $f:Xto Ytimes S^1$ be a continuous map. Show there exists a $g:Xto Y$ such that $f$ is homotopic to the map $(g,1):Xto Ytimes S^1$.



I am fairly confident in the first part of the proof, ie that $|f_*pi_1(X)|<infty$ (although my group theory is rusty). Since $f$ is continuous, it induces a group homomorphism on $pi_1(X)$, so 1) $f_*pi_1(X)simeqpi_1(X)/Ker(f_*)$ and 2) $f_*pi_1(X)leqpi_1(Ytimes S^1)simeqpi_1(Y)times mathbb{Z}$. In particular, (1) gives that $f_*pi_1(X)$ is finite, hence $simeq Htimes {0}$, where $H$ is some subgroup of $pi_1(Y)$, by (2).



The part I am much less sure of (that it is either correct or necessary) is the remainder. Take $p:Ytimesmathbb{R}to Ytimes S^1$ by $p(y,x)=(y,e^{2pi ix})$. This is a covering map, and $p_*pi_1(Ytimesmathbb{R})=pi_1(Y)times{0}$, so $f$ lifts to some $tilde{f}$. Since the image of any closed loop in $X$ is taken to a closed loop that does not wind around $S^1$ (else, we would get a generator for an infinite fundamental group), $tilde{f}$ is homotopic to some $(g,0)$, which maps to $(g,1)$ under $p$.



I have not used the fact that X and Y are CW complexes anywhere, but the only theorem that I know offhand in the context of CW complexes, subsets, and homotopy of functions would be the Homotopy Extension Theorem, which wouldn't make sense here (since $f(X)$ is not necessarily a CW complex).







algebraic-topology homotopy-theory fundamental-groups cw-complexes






share|cite|improve this question















share|cite|improve this question













share|cite|improve this question




share|cite|improve this question








edited Dec 29 '18 at 3:47







Pepper

















asked Dec 28 '18 at 22:47









PepperPepper

508




508












  • $begingroup$
    The map to $Y$ is a red herring, since maps into a product are determined by the maps into the factors. For a fancy proof using cohomology/homology see: math.stackexchange.com/questions/380383/… .
    $endgroup$
    – Justin Young
    Dec 30 '18 at 13:30










  • $begingroup$
    That's a very nice proof, although I'll need to make it to homology in my notes before I can properly parse what's going on.
    $endgroup$
    – Pepper
    Dec 31 '18 at 14:10




















  • $begingroup$
    The map to $Y$ is a red herring, since maps into a product are determined by the maps into the factors. For a fancy proof using cohomology/homology see: math.stackexchange.com/questions/380383/… .
    $endgroup$
    – Justin Young
    Dec 30 '18 at 13:30










  • $begingroup$
    That's a very nice proof, although I'll need to make it to homology in my notes before I can properly parse what's going on.
    $endgroup$
    – Pepper
    Dec 31 '18 at 14:10


















$begingroup$
The map to $Y$ is a red herring, since maps into a product are determined by the maps into the factors. For a fancy proof using cohomology/homology see: math.stackexchange.com/questions/380383/… .
$endgroup$
– Justin Young
Dec 30 '18 at 13:30




$begingroup$
The map to $Y$ is a red herring, since maps into a product are determined by the maps into the factors. For a fancy proof using cohomology/homology see: math.stackexchange.com/questions/380383/… .
$endgroup$
– Justin Young
Dec 30 '18 at 13:30












$begingroup$
That's a very nice proof, although I'll need to make it to homology in my notes before I can properly parse what's going on.
$endgroup$
– Pepper
Dec 31 '18 at 14:10






$begingroup$
That's a very nice proof, although I'll need to make it to homology in my notes before I can properly parse what's going on.
$endgroup$
– Pepper
Dec 31 '18 at 14:10












1 Answer
1






active

oldest

votes


















2












$begingroup$

This is basically correct and is the natural argument to make but your argument at the end is unclear. The reason that $tilde{f}$ is homotopic to some $(g,0)$ is simply that $mathbb{R}$ is contractible, so you can take $tilde{f}$ and contract its second coordinate to $0$ via a homotopy. Your explanation seemed to be mixing this up with the reason that the lift $tilde{f}$ exists at all.



As for where the assumption that the spaces are CW-complexes comes in, you need something about $X$ to be able to say that the lift $tilde{f}$ exists at all (the usual lifting theorems for covering spaces only work for sufficiently nice spaces). In particular, it suffices to know that $X$ is path-connected and locally path-connected, which follows if you know that $X$ is a connected CW-complex. The assumption that $Y$ is a CW-complex is completely unnecessary.



By the way, the fact that $f_*pi_1(X)$ is finite is completely trivial and requires no group theory: it is the image of the finite set $pi_1(X)$ under a function.






share|cite|improve this answer









$endgroup$













  • $begingroup$
    So, in the lifting portion of the argument, it follows as soon as there is a lift to $Ytimesmathbb{R}$? The comments about not winding around $S^1$ were holdovers from my first attempt, and things got a little muddled along the way.Also, thank you for pointing out that I implicitly used X path connected and locally path connected - I had forgotten that that was a requirement for the theorem I used for the existence of a lift.
    $endgroup$
    – Pepper
    Dec 29 '18 at 3:53












  • $begingroup$
    The following seems a much simpler argument, is it incorrect? Let $p: Ytimes S^1to S^1$ be the projection, it follows from $pi_1(X)$ finite that $(pcirc f)_*$ is constant and $pcirc f$ is null-homotop. If $h_t$ is the homotopy contracting the map, then $(f_Y, h_t)$ is a homotopy between $f=(f_Y,f_{S^1})$ and $g=(f_Y,1)$.
    $endgroup$
    – s.harp
    Dec 29 '18 at 18:04










  • $begingroup$
    @s.harp: That's essentially the same argument when you fill in the details. In particular, to prove $pcirc f$ is nullhomotopic you need to lift it to the universal cover. So you're doing the same thing, just separating out the second coordinate of $f$ before lifting to a covering space instead of after.
    $endgroup$
    – Eric Wofsey
    Dec 29 '18 at 18:10












  • $begingroup$
    Ah, you are right.
    $endgroup$
    – s.harp
    Dec 29 '18 at 18:12












Your Answer








StackExchange.ready(function() {
var channelOptions = {
tags: "".split(" "),
id: "69"
};
initTagRenderer("".split(" "), "".split(" "), channelOptions);

StackExchange.using("externalEditor", function() {
// Have to fire editor after snippets, if snippets enabled
if (StackExchange.settings.snippets.snippetsEnabled) {
StackExchange.using("snippets", function() {
createEditor();
});
}
else {
createEditor();
}
});

function createEditor() {
StackExchange.prepareEditor({
heartbeatType: 'answer',
autoActivateHeartbeat: false,
convertImagesToLinks: true,
noModals: true,
showLowRepImageUploadWarning: true,
reputationToPostImages: 10,
bindNavPrevention: true,
postfix: "",
imageUploader: {
brandingHtml: "Powered by u003ca class="icon-imgur-white" href="https://imgur.com/"u003eu003c/au003e",
contentPolicyHtml: "User contributions licensed under u003ca href="https://creativecommons.org/licenses/by-sa/3.0/"u003ecc by-sa 3.0 with attribution requiredu003c/au003e u003ca href="https://stackoverflow.com/legal/content-policy"u003e(content policy)u003c/au003e",
allowUrls: true
},
noCode: true, onDemand: true,
discardSelector: ".discard-answer"
,immediatelyShowMarkdownHelp:true
});


}
});














draft saved

draft discarded


















StackExchange.ready(
function () {
StackExchange.openid.initPostLogin('.new-post-login', 'https%3a%2f%2fmath.stackexchange.com%2fquestions%2f3055358%2fhomotopy-of-continuous-map-from-a-space-with-finite-fundamental-group%23new-answer', 'question_page');
}
);

Post as a guest















Required, but never shown

























1 Answer
1






active

oldest

votes








1 Answer
1






active

oldest

votes









active

oldest

votes






active

oldest

votes









2












$begingroup$

This is basically correct and is the natural argument to make but your argument at the end is unclear. The reason that $tilde{f}$ is homotopic to some $(g,0)$ is simply that $mathbb{R}$ is contractible, so you can take $tilde{f}$ and contract its second coordinate to $0$ via a homotopy. Your explanation seemed to be mixing this up with the reason that the lift $tilde{f}$ exists at all.



As for where the assumption that the spaces are CW-complexes comes in, you need something about $X$ to be able to say that the lift $tilde{f}$ exists at all (the usual lifting theorems for covering spaces only work for sufficiently nice spaces). In particular, it suffices to know that $X$ is path-connected and locally path-connected, which follows if you know that $X$ is a connected CW-complex. The assumption that $Y$ is a CW-complex is completely unnecessary.



By the way, the fact that $f_*pi_1(X)$ is finite is completely trivial and requires no group theory: it is the image of the finite set $pi_1(X)$ under a function.






share|cite|improve this answer









$endgroup$













  • $begingroup$
    So, in the lifting portion of the argument, it follows as soon as there is a lift to $Ytimesmathbb{R}$? The comments about not winding around $S^1$ were holdovers from my first attempt, and things got a little muddled along the way.Also, thank you for pointing out that I implicitly used X path connected and locally path connected - I had forgotten that that was a requirement for the theorem I used for the existence of a lift.
    $endgroup$
    – Pepper
    Dec 29 '18 at 3:53












  • $begingroup$
    The following seems a much simpler argument, is it incorrect? Let $p: Ytimes S^1to S^1$ be the projection, it follows from $pi_1(X)$ finite that $(pcirc f)_*$ is constant and $pcirc f$ is null-homotop. If $h_t$ is the homotopy contracting the map, then $(f_Y, h_t)$ is a homotopy between $f=(f_Y,f_{S^1})$ and $g=(f_Y,1)$.
    $endgroup$
    – s.harp
    Dec 29 '18 at 18:04










  • $begingroup$
    @s.harp: That's essentially the same argument when you fill in the details. In particular, to prove $pcirc f$ is nullhomotopic you need to lift it to the universal cover. So you're doing the same thing, just separating out the second coordinate of $f$ before lifting to a covering space instead of after.
    $endgroup$
    – Eric Wofsey
    Dec 29 '18 at 18:10












  • $begingroup$
    Ah, you are right.
    $endgroup$
    – s.harp
    Dec 29 '18 at 18:12
















2












$begingroup$

This is basically correct and is the natural argument to make but your argument at the end is unclear. The reason that $tilde{f}$ is homotopic to some $(g,0)$ is simply that $mathbb{R}$ is contractible, so you can take $tilde{f}$ and contract its second coordinate to $0$ via a homotopy. Your explanation seemed to be mixing this up with the reason that the lift $tilde{f}$ exists at all.



As for where the assumption that the spaces are CW-complexes comes in, you need something about $X$ to be able to say that the lift $tilde{f}$ exists at all (the usual lifting theorems for covering spaces only work for sufficiently nice spaces). In particular, it suffices to know that $X$ is path-connected and locally path-connected, which follows if you know that $X$ is a connected CW-complex. The assumption that $Y$ is a CW-complex is completely unnecessary.



By the way, the fact that $f_*pi_1(X)$ is finite is completely trivial and requires no group theory: it is the image of the finite set $pi_1(X)$ under a function.






share|cite|improve this answer









$endgroup$













  • $begingroup$
    So, in the lifting portion of the argument, it follows as soon as there is a lift to $Ytimesmathbb{R}$? The comments about not winding around $S^1$ were holdovers from my first attempt, and things got a little muddled along the way.Also, thank you for pointing out that I implicitly used X path connected and locally path connected - I had forgotten that that was a requirement for the theorem I used for the existence of a lift.
    $endgroup$
    – Pepper
    Dec 29 '18 at 3:53












  • $begingroup$
    The following seems a much simpler argument, is it incorrect? Let $p: Ytimes S^1to S^1$ be the projection, it follows from $pi_1(X)$ finite that $(pcirc f)_*$ is constant and $pcirc f$ is null-homotop. If $h_t$ is the homotopy contracting the map, then $(f_Y, h_t)$ is a homotopy between $f=(f_Y,f_{S^1})$ and $g=(f_Y,1)$.
    $endgroup$
    – s.harp
    Dec 29 '18 at 18:04










  • $begingroup$
    @s.harp: That's essentially the same argument when you fill in the details. In particular, to prove $pcirc f$ is nullhomotopic you need to lift it to the universal cover. So you're doing the same thing, just separating out the second coordinate of $f$ before lifting to a covering space instead of after.
    $endgroup$
    – Eric Wofsey
    Dec 29 '18 at 18:10












  • $begingroup$
    Ah, you are right.
    $endgroup$
    – s.harp
    Dec 29 '18 at 18:12














2












2








2





$begingroup$

This is basically correct and is the natural argument to make but your argument at the end is unclear. The reason that $tilde{f}$ is homotopic to some $(g,0)$ is simply that $mathbb{R}$ is contractible, so you can take $tilde{f}$ and contract its second coordinate to $0$ via a homotopy. Your explanation seemed to be mixing this up with the reason that the lift $tilde{f}$ exists at all.



As for where the assumption that the spaces are CW-complexes comes in, you need something about $X$ to be able to say that the lift $tilde{f}$ exists at all (the usual lifting theorems for covering spaces only work for sufficiently nice spaces). In particular, it suffices to know that $X$ is path-connected and locally path-connected, which follows if you know that $X$ is a connected CW-complex. The assumption that $Y$ is a CW-complex is completely unnecessary.



By the way, the fact that $f_*pi_1(X)$ is finite is completely trivial and requires no group theory: it is the image of the finite set $pi_1(X)$ under a function.






share|cite|improve this answer









$endgroup$



This is basically correct and is the natural argument to make but your argument at the end is unclear. The reason that $tilde{f}$ is homotopic to some $(g,0)$ is simply that $mathbb{R}$ is contractible, so you can take $tilde{f}$ and contract its second coordinate to $0$ via a homotopy. Your explanation seemed to be mixing this up with the reason that the lift $tilde{f}$ exists at all.



As for where the assumption that the spaces are CW-complexes comes in, you need something about $X$ to be able to say that the lift $tilde{f}$ exists at all (the usual lifting theorems for covering spaces only work for sufficiently nice spaces). In particular, it suffices to know that $X$ is path-connected and locally path-connected, which follows if you know that $X$ is a connected CW-complex. The assumption that $Y$ is a CW-complex is completely unnecessary.



By the way, the fact that $f_*pi_1(X)$ is finite is completely trivial and requires no group theory: it is the image of the finite set $pi_1(X)$ under a function.







share|cite|improve this answer












share|cite|improve this answer



share|cite|improve this answer










answered Dec 28 '18 at 23:13









Eric WofseyEric Wofsey

193k14221352




193k14221352












  • $begingroup$
    So, in the lifting portion of the argument, it follows as soon as there is a lift to $Ytimesmathbb{R}$? The comments about not winding around $S^1$ were holdovers from my first attempt, and things got a little muddled along the way.Also, thank you for pointing out that I implicitly used X path connected and locally path connected - I had forgotten that that was a requirement for the theorem I used for the existence of a lift.
    $endgroup$
    – Pepper
    Dec 29 '18 at 3:53












  • $begingroup$
    The following seems a much simpler argument, is it incorrect? Let $p: Ytimes S^1to S^1$ be the projection, it follows from $pi_1(X)$ finite that $(pcirc f)_*$ is constant and $pcirc f$ is null-homotop. If $h_t$ is the homotopy contracting the map, then $(f_Y, h_t)$ is a homotopy between $f=(f_Y,f_{S^1})$ and $g=(f_Y,1)$.
    $endgroup$
    – s.harp
    Dec 29 '18 at 18:04










  • $begingroup$
    @s.harp: That's essentially the same argument when you fill in the details. In particular, to prove $pcirc f$ is nullhomotopic you need to lift it to the universal cover. So you're doing the same thing, just separating out the second coordinate of $f$ before lifting to a covering space instead of after.
    $endgroup$
    – Eric Wofsey
    Dec 29 '18 at 18:10












  • $begingroup$
    Ah, you are right.
    $endgroup$
    – s.harp
    Dec 29 '18 at 18:12


















  • $begingroup$
    So, in the lifting portion of the argument, it follows as soon as there is a lift to $Ytimesmathbb{R}$? The comments about not winding around $S^1$ were holdovers from my first attempt, and things got a little muddled along the way.Also, thank you for pointing out that I implicitly used X path connected and locally path connected - I had forgotten that that was a requirement for the theorem I used for the existence of a lift.
    $endgroup$
    – Pepper
    Dec 29 '18 at 3:53












  • $begingroup$
    The following seems a much simpler argument, is it incorrect? Let $p: Ytimes S^1to S^1$ be the projection, it follows from $pi_1(X)$ finite that $(pcirc f)_*$ is constant and $pcirc f$ is null-homotop. If $h_t$ is the homotopy contracting the map, then $(f_Y, h_t)$ is a homotopy between $f=(f_Y,f_{S^1})$ and $g=(f_Y,1)$.
    $endgroup$
    – s.harp
    Dec 29 '18 at 18:04










  • $begingroup$
    @s.harp: That's essentially the same argument when you fill in the details. In particular, to prove $pcirc f$ is nullhomotopic you need to lift it to the universal cover. So you're doing the same thing, just separating out the second coordinate of $f$ before lifting to a covering space instead of after.
    $endgroup$
    – Eric Wofsey
    Dec 29 '18 at 18:10












  • $begingroup$
    Ah, you are right.
    $endgroup$
    – s.harp
    Dec 29 '18 at 18:12
















$begingroup$
So, in the lifting portion of the argument, it follows as soon as there is a lift to $Ytimesmathbb{R}$? The comments about not winding around $S^1$ were holdovers from my first attempt, and things got a little muddled along the way.Also, thank you for pointing out that I implicitly used X path connected and locally path connected - I had forgotten that that was a requirement for the theorem I used for the existence of a lift.
$endgroup$
– Pepper
Dec 29 '18 at 3:53






$begingroup$
So, in the lifting portion of the argument, it follows as soon as there is a lift to $Ytimesmathbb{R}$? The comments about not winding around $S^1$ were holdovers from my first attempt, and things got a little muddled along the way.Also, thank you for pointing out that I implicitly used X path connected and locally path connected - I had forgotten that that was a requirement for the theorem I used for the existence of a lift.
$endgroup$
– Pepper
Dec 29 '18 at 3:53














$begingroup$
The following seems a much simpler argument, is it incorrect? Let $p: Ytimes S^1to S^1$ be the projection, it follows from $pi_1(X)$ finite that $(pcirc f)_*$ is constant and $pcirc f$ is null-homotop. If $h_t$ is the homotopy contracting the map, then $(f_Y, h_t)$ is a homotopy between $f=(f_Y,f_{S^1})$ and $g=(f_Y,1)$.
$endgroup$
– s.harp
Dec 29 '18 at 18:04




$begingroup$
The following seems a much simpler argument, is it incorrect? Let $p: Ytimes S^1to S^1$ be the projection, it follows from $pi_1(X)$ finite that $(pcirc f)_*$ is constant and $pcirc f$ is null-homotop. If $h_t$ is the homotopy contracting the map, then $(f_Y, h_t)$ is a homotopy between $f=(f_Y,f_{S^1})$ and $g=(f_Y,1)$.
$endgroup$
– s.harp
Dec 29 '18 at 18:04












$begingroup$
@s.harp: That's essentially the same argument when you fill in the details. In particular, to prove $pcirc f$ is nullhomotopic you need to lift it to the universal cover. So you're doing the same thing, just separating out the second coordinate of $f$ before lifting to a covering space instead of after.
$endgroup$
– Eric Wofsey
Dec 29 '18 at 18:10






$begingroup$
@s.harp: That's essentially the same argument when you fill in the details. In particular, to prove $pcirc f$ is nullhomotopic you need to lift it to the universal cover. So you're doing the same thing, just separating out the second coordinate of $f$ before lifting to a covering space instead of after.
$endgroup$
– Eric Wofsey
Dec 29 '18 at 18:10














$begingroup$
Ah, you are right.
$endgroup$
– s.harp
Dec 29 '18 at 18:12




$begingroup$
Ah, you are right.
$endgroup$
– s.harp
Dec 29 '18 at 18:12


















draft saved

draft discarded




















































Thanks for contributing an answer to Mathematics Stack Exchange!


  • Please be sure to answer the question. Provide details and share your research!

But avoid



  • Asking for help, clarification, or responding to other answers.

  • Making statements based on opinion; back them up with references or personal experience.


Use MathJax to format equations. MathJax reference.


To learn more, see our tips on writing great answers.




draft saved


draft discarded














StackExchange.ready(
function () {
StackExchange.openid.initPostLogin('.new-post-login', 'https%3a%2f%2fmath.stackexchange.com%2fquestions%2f3055358%2fhomotopy-of-continuous-map-from-a-space-with-finite-fundamental-group%23new-answer', 'question_page');
}
);

Post as a guest















Required, but never shown





















































Required, but never shown














Required, but never shown












Required, but never shown







Required, but never shown

































Required, but never shown














Required, but never shown












Required, but never shown







Required, but never shown







Popular posts from this blog

Bundesstraße 106

Ida-Boy-Ed-Garten

Le Mesnil-Réaume